1
$\begingroup$

Suppose $X$ is compact Hausdorff and $\mathcal P$ is a partition of $X$. Define the map $\pi:X \to \mathcal P$ taking each point to the unique partition element containing it. Give $\mathcal P$ the quotient topology under $\pi$. Assume the quotient topology is Hausdorff. This is equivalent to assuming $\pi$ is a closed map, and also equivalent to the condition that ($*$) every partition element is contained in an open subset of $X$ that is a union of partition elements.

Now take a closed set $Y \subset X$ which is a union of partition elements. There are two ways to put a topology on $\mathcal P_Y = \{P \in \mathcal P: P \subset Y\}$.

  1. Consider the subspace $\pi(Y) \subset \mathcal P$

  2. Give $\mathcal P_Y$ the quotient topology under the natural map $\Pi: Y \to \mathcal P_Y$.

Condition $(*)$ for $\mathcal P$ implies the same condition for $\mathcal P_Y$, and so the quotient topology (2) is Hausdorff. Since the two maps have the same fibers $-$ precisely the elements of $\mathcal P_Y$ $-$ it follows from the fact that surjections between compact Hausdorff spaces are quotient maps that the two topologies are natutally homeomorphic.

I suspect the above fails horribly in case the subspace $Y$ is not closed. However I am having trouble coming up with an example. Has anyone considered the above situation before?

$\endgroup$
2
  • $\begingroup$ There is something wrong in your last statement in the first paragraph, because every partition element is always contained in $X$ which is an open subset of $X$ and the union of all partition elements. So the equivalence would mean that $\pi$ is always closed. $\endgroup$ Commented Nov 16 at 11:00
  • $\begingroup$ I'm not entirely sure, but if you just consider $Y$ as being the union of elements of the partition, you can prove that $\pi^{-1}(\pi(Y)) = Y$ and from there that the restriction $\pi\colon Y\to\pi(Y)$ is a quotient map. Now you have two quotient maps on $Y$ which have the same fibers. Each one induces a continuous map via the other, which (I guess) are inverse of each other. However, this is just a hint that comes to mind. $\endgroup$ Commented Nov 16 at 11:10

1 Answer 1

3
$\begingroup$

The quotients you are considering are given by upper semi-continuous decompositions.

The subspace and quotient topology will coincide if the quotient map $\pi$ is closed.

See this answer of mine where I redirect you to theorem in Topology by Dugundji.

In particular, since your map $\pi:X\to \mathcal{P}$ is closed, the subspace topology on $\mathcal{P}_Y$ and quotient topology from the map $\pi\restriction_Y:Y\to\mathcal{P}_Y$ will coincide.

$\endgroup$
1
  • $\begingroup$ This is exactly what I was looking for. You even corrected my faulty definition. Thanks a million! $\endgroup$ Commented 2 days ago

You must log in to answer this question.

Start asking to get answers

Find the answer to your question by asking.

Ask question

Explore related questions

See similar questions with these tags.